The Hardest Integral From The Hardest Test (Putnam Exam)

Поделиться
HTML-код
  • Опубликовано: 7 сен 2024
  • 🎥 The Putnam's Toughest Integral Solved! 📚🔥
    Join me as I tackle one of the most challenging integrals from the Putnam Exam: the infamous integral x^-x ! In this video, we'll break down the problem step-by-step. Whether you're a math enthusiast, a student preparing for the Putnam Exam, or just curious about advanced calculus, this video is for you!
    🔍 What You'll Learn:
    The Putnam Exam and its significance
    Different approaches to solving integrals
    Detailed solution to the integral x^-x
    Tips and tricks for solving complex math problems.
    👍 Don't forget to like, comment, and subscribe for more math content!
    🔔 Hit the bell icon to get notified whenever I upload new videos.
    📢 Share this video with your friends who love math and challenges!
    📱 Follow me on social media:
    Instagram: @jagosprivatelife
    📧 Contact me for collaboration or inquiries
    🔍 Relevant Hashtags:
    #PutnamExam #IntegralCalculus #MathChallenge #AdvancedMath #Calculus #Mathematics #PutnamCompetition #MathTutorial #LearnMath #STEM #Education #MathVideo
    Thank you for watching, and happy solving!

Комментарии • 207

  • @Jagoalexander
    @Jagoalexander  Месяц назад +37

    Thank you for 1k subscribers! Onwards and upwards.

  • @OliverGoodman-todd
    @OliverGoodman-todd Месяц назад +184

    that guy definitely had anger issues haha

    • @knowledgestar1081
      @knowledgestar1081 Месяц назад +4

      People don't have ability to make videos like this RUclipsr so they think that using such great words makes them powerfull.😂 Actually people who lack vocabulary start using such words.

    • @knowledgestar1081
      @knowledgestar1081 Месяц назад +3

      Whatever you do people have some reason to scold you

  • @naturallyinterested7569
    @naturallyinterested7569 Месяц назад +89

    The final value is roughly 1.3

    • @zachariastsampasidis8880
      @zachariastsampasidis8880 Месяц назад +8

      Should be transcendental due to Liouville's theorem

    • @Lily_Kuroki
      @Lily_Kuroki Месяц назад +8

      1.29129 actually which is also pretty elegant

    • @Mathslover666
      @Mathslover666 Месяц назад +6

      It's 1.29128599706....approx I graph it on desmos. Lol

    • @Mathslover666
      @Mathslover666 Месяц назад +2

      ​@@zachariastsampasidis8880 common sense bro 😗

    • @Kainnable
      @Kainnable День назад

      NIntegrate[x^-x, {x,0,1}] = 1.29129

  • @jujdj6214
    @jujdj6214 Месяц назад +53

    i think thats the best video for an introduction to your channel i could get

  • @SlightSmile
    @SlightSmile Месяц назад +20

    I love the "complaining and explanation" that people seem to complain about. Just keep doing your thing lol.
    Very nice solution btw.

    • @Jagoalexander
      @Jagoalexander  Месяц назад

      Thanks mate !

    • @colt4667
      @colt4667 12 дней назад

      I wish his penmanship were a little better.

  • @PrometheusZandski
    @PrometheusZandski Месяц назад +5

    You are a gentleman and a scholar, sir. Apology accepted. I personally don't want music to distract my thought process when doing math.
    Absolutely amazing solution.

  • @bontear2628
    @bontear2628 Месяц назад +52

    that comment part felt more like a chess review

  • @awkwardhamster8541
    @awkwardhamster8541 Месяц назад +23

    whys ur comment section filled with weird ppl ? ur videos r very well explained and ur voice is very calming :)

    • @DeadJDona
      @DeadJDona Месяц назад

      @@awkwardhamster8541 dats y we here

  • @dalisabe62
    @dalisabe62 8 дней назад +1

    When I first saw the problem, I was tempted to use the definition of the integral as the limit of the sum as the index goes to infinity. It’s nice to see at least that the concept of integration is the limit of an infinite sum of a discrete sum.

  • @johnlv12
    @johnlv12 Месяц назад +16

    There is exactly one "k" in the video- at 12:58. I was able to follow everything easily because you said everything as you were writing. I just point it out because I thought it was funny that one time you realized the "k" looks like a "u" and made a special effort to make it look good, but immediately reverted back after that. Everything is good.
    On a separate issue, I don't doubt that this is the solution that would get 10/10 points on the exam, but to me it doesn't look any simpler than the original integral. They are both infinite sums that have to be evaluated numerically. The solution converges quickly and even the first handful of terms are good enough to approximate the entire sum, so that's good if you were trying to get a numerical approximation. My problem is with the statement of the problem- it could have said show that the integral equals the sum.

    • @Hand_of_the_king
      @Hand_of_the_king Месяц назад

      There is one at 18:36 too

    • @Jagoalexander
      @Jagoalexander  Месяц назад +3

      😭😭😭😭 I think this is because it's just been exam season so I got used to just writing really fast and not caring about handwriting

  • @ShenghuiYang
    @ShenghuiYang Месяц назад +93

    Sophomore's dream.

    • @sydhenderson6753
      @sydhenderson6753 Месяц назад +3

      And true to life, I proved it (and the corresponding integral with x^x) in my sophomore year.

  • @txikitofandango
    @txikitofandango Месяц назад +84

    Nice video but i thought it could use more music to jazz it up

    • @harshnambiar
      @harshnambiar Месяц назад

      Maybe a lot of people would have downvoted if the music was annoying

    • @gdcuaer4076
      @gdcuaer4076 Месяц назад

      Yea we need to jazz this vid up!!!!!!
      Would really help

  • @dalisabe62
    @dalisabe62 8 дней назад +1

    Seeing limited integrals as the infinite sum of a converging discrete sequence helps conceptualize the process incorporated in this solution. After all, the only difference between the final left and right hand in the answer is the shift of the integration symbol to the summation symbol and the change of the limits of integration and summation (from zero to infinity) to (one to infinity) respectively. That makes perfect sense because X to the power of negative X has undetermined value when X = 0 or = 1 by definition. Of course, the original problem was written in terms of X not alpha, so the back substitution needs to be made.

    • @chibuzorbethel3292
      @chibuzorbethel3292 6 дней назад

      Exactly what I'm thinking; back substitution to return the result in terms of the original variable x

  • @cyrilanderson
    @cyrilanderson Месяц назад +2

    Nice. I think Michael Penn did one on this integral too.
    It looks like it would converge extremely quickly too, which would make it easy to approximate to a chosen level of precision.

  • @willnewton2247
    @willnewton2247 Месяц назад +10

    awesome vid! now i just have to go learn about the gamma function lol

  • @LouisEmery
    @LouisEmery Месяц назад +42

    how would a student know when to stop? The final result looks like an intermediate result as well.

    • @declandougan7243
      @declandougan7243 Месяц назад +9

      Intuition and experience. This isn't an entrance exam or something where enough study is supposed to allow anyone with sufficient time to ace it. It's the Putnam exam, an extremely prestigious and entirely optional endeavor.

    • @LouisLeCrack
      @LouisLeCrack Месяц назад +2

      It’s relatively evident that you can’t simplify the sum. If not you probably don’t have the level to pass this exam

    • @aisolutionsindia7138
      @aisolutionsindia7138 Месяц назад

      @@LouisLeCrack are you sure a more elegant expression doesnt exist "at your level"

    • @winstongludovatz111
      @winstongludovatz111 Месяц назад

      The series converges very fast, therefore numerically useful.

  • @faramarzsarmadi2398
    @faramarzsarmadi2398 26 дней назад +1

    At x = 0, we have 0^0, which is undefined.

  • @quite_unknown_1
    @quite_unknown_1 Месяц назад +2

    I came for the math, but I enjoyed the analysis of dashxdr’s comment the most

  • @Khashayarissi-ob4yj
    @Khashayarissi-ob4yj Месяц назад +3

    With luck and more power to you.
    hoping for more videos.

  • @bigchungusdriplord2301
    @bigchungusdriplord2301 Месяц назад +5

    You're the perfection of passive agressive lmao

    • @BridgeBum
      @BridgeBum Месяц назад

      I was thinking how the contrast between the thanks for all the constructive comments and the selected comment cussing him out was a textbook example of British dry humo(u)r. Chef's kiss.

  • @jessewolf7649
    @jessewolf7649 Месяц назад +8

    You need a justification for interchanging the infinite sum and the integral at 6:50.

    • @attica7980
      @attica7980 Месяц назад +2

      The series is uniformly convergent on [0,1], so the interchange of the sum and integral is OK.

    • @bantix9902
      @bantix9902 Месяц назад +2

      ​@@attica7980 gotta prove that as well 😂

    • @attica7980
      @attica7980 Месяц назад

      @@bantix9902 x ln x has absolute value less than 1 in [0,1], and the sum of 1/n! is convergent; hence the function series is dominated in absolute value by a convergent numerical series, so it is uniformly convergent.

  • @user-vu5ni9ux1i
    @user-vu5ni9ux1i Месяц назад +3

    An awsome video it is! I wish your k and u (and possibly n) would be more distinctive.

  • @faramarzsarmadi2398
    @faramarzsarmadi2398 26 дней назад +1

    Given the behavior at x = 0, we can't integrate from exactly 0. We need to use a limit:
    lim(ε→0+) ∫(ε to ∞) x^(-x) dx

  • @improbabilty
    @improbabilty 23 дня назад +1

    im new but i think your voice is really great!!!!
    no music is no problem!!

    • @Jagoalexander
      @Jagoalexander  23 дня назад +1

      Thank you, how nice :)

    • @improbabilty
      @improbabilty 23 дня назад

      @@Jagoalexander you're welcome 🤗
      you got a new subscriber and few views

  • @Dissimulate
    @Dissimulate 28 дней назад

    There is a shorter path between these forms because both are equal to the limit as m goes to infinity of (sum k=1 to m ( ((k/m)^(-k/m))/m ). The 1/m represents the width of each segment of a partial sum in the integral.

  • @attica7980
    @attica7980 Месяц назад +3

    I knew the final equation in the video about the integral being equal to the sum. But how does that answer the question as to what the integral is. It would be good to know how the question was asked, since as stated (find the integral), one cannot know whether the final equation indeed solves the problem.

    • @krishpandey854
      @krishpandey854 Месяц назад

      I have encountered this question before in a test in India. In that question, they asked the integer value less than or equal to the integral of x^x. So, you had to derive this result (we used reduction because gamma function isn't taught at high school level) and then calculate the first 4 terms to get the approximate value as 129 or something like that

  • @nirjharchaudhuri6484
    @nirjharchaudhuri6484 Месяц назад +4

    I thought the music was incredibly beautiful

  • @wolfgangroth6265
    @wolfgangroth6265 28 дней назад

    Thank you for the video. Question for all: Is there any way (similar or not) to calculate the same Integral from 1 to infinity? It must be konvergent, because for X>2 the function x^-x < x^-2 so the Integral is smaller than the Integral of x^-2 wich is konvergent but how to calculate? I’m looking for this for decades…

  • @GrandAdmiralMitthrawnuruodo
    @GrandAdmiralMitthrawnuruodo Месяц назад

    Absolutely incredible. I love those Integrals. Thanks! I‘m surprised that I could solve this with my High school knowledge. :D

  • @lucascardoso5332
    @lucascardoso5332 Месяц назад

    That's funny. I solved this problem while in my second year of undergrad. I was at my friend's place and we had ordered pizza. He had just finished taking his first course on Calculus and I joked that he couldn't find the derivative of x^x, which he did. Then he challenged me to find the integral, which I did too. It was fun!

  • @alexbenanti5125
    @alexbenanti5125 Месяц назад +1

    I COULD BARELY HEAR THE MUSIC OVER YOU SOLVING THAT INTEGRAL. SO OBNOXIOUS!!!

  • @radiomanroud1559
    @radiomanroud1559 Месяц назад +1

    Absolutely brillant ! Thank you a lot

  • @rob876
    @rob876 Месяц назад

    I liked the music in your last video. I didn't think it was too loud. I thought it was quite calming.
    I, however, did not like your choice of indexing variable in your final answer in this video. Why not just pick n?
    Great video - very clearly explained. Keep up the good work.

  • @zakimoutahir1055
    @zakimoutahir1055 Месяц назад +4

    Amazing video thanks a lot. I don't know if you know it but there is another integral that's look basically the same called "bernoulli integral", with the only deffencies being the negativ sign in the power. (No worries about the music we all make mistake 🫡🎉)

  • @BramCohen
    @BramCohen 28 дней назад

    There's a strange vagueness to the question here. How do you know that that answer is the final answer and can't be 'simplified' further? What functions would be allowed in that 'simplification'?

  • @well_said7846
    @well_said7846 Месяц назад

    The final result is mind blowing great work.😊

  • @alwaysfourfun1671
    @alwaysfourfun1671 22 дня назад

    What does the final summation converge to, e.g. according to Wolfram? Very nice video.

  • @jimcoughlin4057
    @jimcoughlin4057 29 дней назад

    I believe you must justify pulling the summation out of the integral. It is not enough to simply say that the sum (-1)^k/n! is a constant; k still appears in the integrand and these 2 operations do not always commute.

  • @B00MBASTIC
    @B00MBASTIC Месяц назад +1

    Where’s the Family Guy and the Soap Cutting videos on the sides?
    Please consider adding them next video!

  • @bryanfranco3943
    @bryanfranco3943 29 дней назад

    Wouldn't your result mean that the area from 0 to 1 and from 1 to infinity are the same since alpha and x can be taken to be anything, even eachother?? Or am I wrong because you've introduced several changes of variables that places constraints on alpha?

  • @hamoudiayoub9341
    @hamoudiayoub9341 Месяц назад

    Integral is a continuous sum , we solve it by finding the value of that sum but you transfer it to a discreet sum

  • @thomaslangbein297
    @thomaslangbein297 Месяц назад +7

    Don’t complain, don’t explain. Just keep to the topic.

  • @looney1023
    @looney1023 Месяц назад +1

    Is this actually a Putnam question? What's the problem statement? It's a very cool result but how would a Putnam test taker know that the answer is simplified enough at the end, since it can't be evaluated exactly?

    • @Jagoalexander
      @Jagoalexander  Месяц назад

      The question was show the integral can be written as the sum

  • @skilz8098
    @skilz8098 Месяц назад

    Perhaps rewrite this integral as being the integration from 0 to 1: x^(i^2 * x) dx
    Maybe this might help. We can think of the exponent: i^2 * x as being periodic where x is the multiple or enumerated counts of a rotation by 180 degrees or PI radians.
    If we look at i^2 as being sqrt(-1) * sqrt(-1) = -1. We know that the value of -1 is a 180-degree rotation of 1. We can use this along with the trigonometric properties to help solve this.
    From what I can see, there is no one exact result from this definite integral. In other words, it's not going to give you a discrete area, volume or region under a curve or manifold. It appears that it is going to give you a series or sequence of them with a specific periodicity determined by the magnitude of x within the exponential component.

  • @JiWon-g1s
    @JiWon-g1s 7 часов назад +1

    For the music in your videos, here's the reality: if it ADDS to the video, then it's good. If it DETRACTS (as it does in your case) then it is a smarmy waste of time.

  • @peterwang6474
    @peterwang6474 Месяц назад

    Ingenious! I wonder if the serial is convergent and if yes, what's the final result?

  • @winstonridgewayhardy
    @winstonridgewayhardy Месяц назад

    I am happy there is no music (but no need to be so angry imho). Wonderfully explained but just a gentle constructive comment is maybe try to write a little more clearly. I know what is going on if I listen along but if I am just looking at the integral I start thinking what is planck's constant popping up in inappropriate place. Haha - just kidding - but you wrote the k really well once so I know you can do it :)

    • @Jagoalexander
      @Jagoalexander  Месяц назад

      Yes this is a common theme, I'll wrote more clearly from now on 😅

  • @Vivir_Astucia
    @Vivir_Astucia Месяц назад +2

    I didn't knew we could take out summation out of integrals.....are there conditions for this?

    • @edreeves6440
      @edreeves6440 Месяц назад

      Absolutely, there are conditions, but if it all works out in the end those conditions probably held.

  • @jayxone
    @jayxone Месяц назад

    Bro had me so confused with the u’s and k’s looking the exact same

  • @VeteranVandal
    @VeteranVandal Месяц назад +1

    That guy doesn't know the internet is for free.

  • @paultaufalele1821
    @paultaufalele1821 Месяц назад

    Woa very cool result. Awesome vid

  • @mauriziofenderico8348
    @mauriziofenderico8348 Месяц назад

    Thank you for the excellent demonstration and...NO MUSIC is perfect so one can really concentrate🙂😂

  • @CPILLOW
    @CPILLOW Месяц назад +4

    great vid!!!

  • @user-yh3yz7kt5e
    @user-yh3yz7kt5e Месяц назад +1

    Praise be to God, mathematics is one of my best subjects. I got a score of 97 in the final exam, and the questions were very difficult. I am proud of my score because my students were crying because of the difficulty of the questions. I was hoping to get a score of 100, but I thank God for what I got. ❤

    • @baselinesweb
      @baselinesweb Месяц назад

      And we need to know this why? Is your ego sated now?

  • @dieguitomvp8338
    @dieguitomvp8338 Месяц назад

    Easy, full series and substitution

  • @hsfjw
    @hsfjw Месяц назад +1

    Use Lambert W function maybe faster?🤔

  • @IcyGM
    @IcyGM Месяц назад

    I loved the video and everything, but there's one thing that I would like clarification about. Isn't there any condition on the function that needs to be met in order to interchange the integral and the summation? And if so, was it satisfied here?
    Thank yih

    • @Jagoalexander
      @Jagoalexander  Месяц назад

      Check my latest video, where I go into detail about this
      I Solved the Impossible Bernoulli Integral!
      ruclips.net/video/T-1fFR8Nk6A/видео.html

  • @sachin251998
    @sachin251998 Месяц назад +4

    the way you write k can be interpreted as k, u or even h lol

  • @tfg601
    @tfg601 20 дней назад

    I find it funny how the first step into finding the derivative and integral of x^-x are the same

  • @edreeves6440
    @edreeves6440 Месяц назад

    Really interesting result but a laboured path through a lot of obvious steps.

    • @Jagoalexander
      @Jagoalexander  Месяц назад

      Which steps are obvious - I wanted to provide a digestible solution to the average viewer who may not have advanced maths classes in their repertoire

  • @shreebhattacharjee3502
    @shreebhattacharjee3502 Месяц назад +3

    amazing!!

  • @hackergaming6869
    @hackergaming6869 Месяц назад +2

    wouldnt you have to resubstitute in whatever alpha is to get the real result?

    • @joeythreeclubs
      @joeythreeclubs Месяц назад +1

      No, alpha is explicitly defined by the infinite sum's bounds

  • @marcnordin9240
    @marcnordin9240 Месяц назад +1

    Great video
    Very inspiring

  • @jakeaustria5445
    @jakeaustria5445 Месяц назад +2

    Thank you

  • @aspirinforbacteria11
    @aspirinforbacteria11 Месяц назад

    Somehow youtube recommended this video to me and out of curiosity I opened it. Then I realized that this is the same person whom I learned Feynman Technique from lol! Subscribed!!

  • @florinleonardpetrescu4677
    @florinleonardpetrescu4677 Месяц назад +1

    what is sum(0, inf)pow(alfa, -alfa) ? you're just gonna live it like that?

    • @BridgeBum
      @BridgeBum Месяц назад

      I suspect there is no closed form answer, but since we are providing feedback: a screenshot to wolframalpha, desmos or the like showing a numerical approximation is a great way to end these type problems.

  • @SeekingTheLoveThatGodMeans7648
    @SeekingTheLoveThatGodMeans7648 Месяц назад

    When I actually went to school, I was quite bothered by things that were distracting or incomplete, for a grade was going to come to hold me responsible for it just the same. Now, nobody is grading me on following it. Still, some others may be trying to keep up with school, and bringing their concerns with them, and perhaps a MITE irritated with the whole thing, because a grade is going to result at the end. I will not cuss you out, but I will urge you to maximize the helpfulness. k is k. It is not h, it is not u. And if on top of it, some random music is coming, this makes it harder. (Turning the sound off is not a good option, for you are speaking.) Thank you very much for understanding. It's not all about you either.

  • @DeadJDona
    @DeadJDona Месяц назад +2

    1:23 nice voic3 btw

  • @on_God_
    @on_God_ Месяц назад

    I actually liked the music, it jus doesn't have to be loud

  • @imarealperson-5412
    @imarealperson-5412 Месяц назад

    Good video, but it could use some music. Your soothing voice would work greatly with some jazz or something. Keep up the good content!

  • @vikivanov5612
    @vikivanov5612 Месяц назад

    Nice final formula

  • @TheFinav
    @TheFinav Месяц назад

    Very nice presentation. Thanks.

  • @ethanbartiromo2888
    @ethanbartiromo2888 Месяц назад +1

    So, a question I have is was the question on the Putnam in the form of “prove that \int_0^1 x^{-x}= \sum_{k = 1}^{\infty} k^{-k}”? Otherwise, I don’t think this would be a valid solution to the problem because it still isn’t in a constant form, it’s in the form of an infinite sum, which still needs different methods to solve/approximate. I took the Putnam twice, once in my freshman year of college, and again in my junior year, and unless it was worded like that, this would probably get a 1 or 4 or something like that, not a full 10

    • @Jagoalexander
      @Jagoalexander  Месяц назад +2

      Yes, the question was show that the integral can be written as the infinite sum. The infinite sum converges very quickly to around 1.2… not an exact value unfortunately

  • @larzcaetano
    @larzcaetano 21 день назад +1

    What kind of sorcery is this

  • @Bodyknock
    @Bodyknock Месяц назад

    What was the actual question in the exam? Maybe I missed it but I don’t see the full question in the video or a link to it in the description. Was the question “Show (this integral) equals (this sum)”?

    • @Jagoalexander
      @Jagoalexander  Месяц назад +2

      Yes it was, the actual question was show it's equal to the sum. Sorry for not explaining that well in the video. I was more focused on the method of solving the integral, the sum converges very quickly to around 1.2..

  • @eluoh
    @eluoh Месяц назад +1

    no music is fine thanks you !

  • @user-fz9go8pj4t
    @user-fz9go8pj4t Месяц назад

    Actually you didn't explain why you could interchange the integral and the sum. This is due to the dominated convergence theorem. We must explain all the details why we can do it

  • @katiekawaii
    @katiekawaii Месяц назад

    That's one of the more bizarre comments I've ever seen.

  • @ghostsofbeauty.9346
    @ghostsofbeauty.9346 Месяц назад +2

    3..

  • @blablub1375
    @blablub1375 Месяц назад +9

    Changing the order of summation and integration can't be done carelessly, there should have been made some rigorous argument why this holds here.

    • @Jagoalexander
      @Jagoalexander  Месяц назад +19

      It's called "proof by knowing that it can be done"

    • @xenmaifirebringer552
      @xenmaifirebringer552 Месяц назад +3

      ​@@Jagoalexander ah, my favourite kind of proof

    • @DeadJDona
      @DeadJDona Месяц назад +2

      @@xenmaifirebringer552 it's like breaking a 4th wall, third time is a charm )

    • @LouisLeCrack
      @LouisLeCrack Месяц назад +1

      Yup that’s the only hard part, the rest is really easy

    • @LouisLeCrack
      @LouisLeCrack Месяц назад

      @@Jagoalexanderyou’re bad bro 😂

  • @uvogin3143
    @uvogin3143 Месяц назад +1

    All of this to write 2 times the same stuff

  • @yaozhang1687
    @yaozhang1687 Месяц назад

    Can you solve the integral from 1 to infinity of x^-x?

    • @numbers93
      @numbers93 Месяц назад

      probably not. You can't swap the infinite sum for the integral if the upper limit of integration is at infinity

    • @yaozhang1687
      @yaozhang1687 Месяц назад

      @@numbers93 Of cause, you can't solve this integral by infinite sum. But this integral definitely converge.(Because 0 < x^-x < e^-x when x > e, and the integral from e to infinity of e^-x converge) But I can't find any method to calculate the exact value of this integral.

  • @brickie9816
    @brickie9816 Месяц назад +1

    this video on the other hand had no music. how outrageous. i can't believe you left out such a crucial element of a math video. because of lack of background music, i don't even know if i can call the video informative. i hope you will rethink your life choices that led you to choose to not put music over your video. literally unwatchable
    edit: i skipped the intro and missed you saying there will be no music 😭 not sure if that makes things

  • @sivaramakrishnanm9816
    @sivaramakrishnanm9816 22 дня назад

    😂well done

  • @DS-wm6rn
    @DS-wm6rn Месяц назад

    I would say the problem solved in the video looks harder than the question asked in the actual exam.
    Namely, asking: "find this integral " does not give you any clues. On the other hand, asking: "prove that this integral equals to this sum" is quite a clue. So the video is a bit misleading/clickbaiting.

  • @VarunDoesMaths
    @VarunDoesMaths Месяц назад

    Nice! What mic and software do you use btw?

  • @moonwatcher2001
    @moonwatcher2001 Месяц назад

    ❤ awesome

  • @krishnamaity5056
    @krishnamaity5056 Месяц назад

    What answer did official Putnam authority give. Was it this one?

  • @marcnordin9240
    @marcnordin9240 Месяц назад

    Which software do you use for writing down your formulae?

  • @arekkrolak6320
    @arekkrolak6320 Месяц назад

    Putting music is fine as long as you are a musician and not talk over it :)

  • @Nip403
    @Nip403 Месяц назад

    Nice analysis of the comment lmao
    At any rate, your voice is already music to my ears

  • @nealkuril3588
    @nealkuril3588 Месяц назад

    4:50

  • @ezu8501
    @ezu8501 Месяц назад

    very cool

  • @DeadJDona
    @DeadJDona Месяц назад +1

    9:00 why'd u keep saying k when writing n? or write n each time you say k?
    geez writing is difficult (c) 3b1b

    • @DeadJDona
      @DeadJDona Месяц назад

      13:00 k

    • @Jagoalexander
      @Jagoalexander  Месяц назад +3

      Both n and k are common variables to use with sums, however I've opted to use k as my variable I'm this video. In the video I do write the letter k but my handwriting makes it look like an n, making it confusing. But to clarify, I'm both saying and writing k

    • @DeadJDona
      @DeadJDona Месяц назад

      @@Jagoalexander it would be easier i guess to just say n ))

    • @_Heb_
      @_Heb_ Месяц назад +1

      @@DeadJDonaBut then you'd be writing k and saying n!

    • @DeadJDona
      @DeadJDona Месяц назад

      @@_Heb_ how would you know that he's writing k then? )) or k factorial?

  • @MrNibiru2112
    @MrNibiru2112 Месяц назад

    Im afraid, Factorization of the constant at 6:26 is not correct

  • @idkusername2981
    @idkusername2981 Месяц назад

    Hey, I see many comments about the handwriting, but it's completely understandable if you follow along, (I have terrible handwriting too).

  • @fderty4
    @fderty4 Месяц назад

    I could solve this integral in the bat of an eye. I chose not to 😎

  • @mau9639
    @mau9639 Месяц назад +2

    12:35 why do they cancel out 😭

    • @Jagoalexander
      @Jagoalexander  Месяц назад +2

      Because when you factor bring the (-1)^k from inside the integral, outside, it combines with the one already there forming (-1)^2k, for any value of k, 2k is even and hence (-1)^2k will always yield 1. Hence we don't need to write it anymore as they reduce to 1

    • @mau9639
      @mau9639 Месяц назад

      @@Jagoalexander omg thanks

  • @cdkw2
    @cdkw2 Месяц назад +1

    Use pokemon music please

  • @Chiavaccio
    @Chiavaccio Месяц назад

    👏👏😱👍